Use app×
Join Bloom Tuition
One on One Online Tuition
JEE MAIN 2025 Foundation Course
NEET 2025 Foundation Course
CLASS 12 FOUNDATION COURSE
CLASS 10 FOUNDATION COURSE
CLASS 9 FOUNDATION COURSE
CLASS 8 FOUNDATION COURSE
0 votes
813 views
in Mathematics by (94.6k points)
closed by
Find the set of all possible real value of a such that the inequality `(x-(a-1))(x-(a^2+2))<0` holds for all `x in (-1,3)dot`<br>A. `(0, 1)`
B. `(infty, -2]`
C. `(-infty, - 1)`
D. `(1, infty)`

1 Answer

0 votes
by (97.5k points)
selected by
 
Best answer
Correct Answer - 2
We have,
` a - 1 le - 1 `
and ` a^(2) + 2 ge 3 `
`a le 0 and a^(2) ge 1`
Hence, a ` le - 1`
image

Welcome to Sarthaks eConnect: A unique platform where students can interact with teachers/experts/students to get solutions to their queries. Students (upto class 10+2) preparing for All Government Exams, CBSE Board Exam, ICSE Board Exam, State Board Exam, JEE (Mains+Advance) and NEET can ask questions from any subject and get quick answers by subject teachers/ experts/mentors/students.

Categories

...